adding lora a_star run

This commit is contained in:
emilyworks 2024-12-12 03:13:24 +00:00
parent 1bd0605949
commit c3080dd586
3 changed files with 359 additions and 5 deletions

View File

@ -0,0 +1,351 @@
,gt,pred
0,2,"The graph of $y=\frac{2}{x^2+x-6}$ has $\boxed{2}$ vertical asymptotes at x=2 and x=-3.
The final answer is 2."
1,10,"The difference between 36 and 26 is $\boxed{10}$.
The final answer is: $\boxed{10}$"
2,i,"The first term $i^5$ can be evaluated by rewriting the exponent as a sum of exponents of 4. Since 5 is equal to $4 \cdot 1 + 1$, $i^5 = (i^4)^1 \cdot i = 1^1 \cdot i = i$.
## Step 1: Evaluate the first term
The first term $i^5$ can be evaluated by rewriting the exponent as a sum of exponents of 4. Since 5 is equal to $4 \cdot 1 + 1$, $i^5 = (i^4)^1 \cdot i = 1^1 \cdot i = i$.
## Step 2: Evaluate the second term
The second term $i^{-25}$ can be evaluated by rewriting the exponent as a sum of exponents of 4. Since 25 is equal to $4 \cdot 6 + 1$, $i^{-25} = (i^4)^{-6} \cdot i^{-1} = 1^{-6} \cdot i^{-1} = 1^{-6} \cdot (i^4)^{-1} \cdot i = 1^{-6} \cdot 1^{-1} \cdot i = 1^{-6} \cdot i = i^{-1}$.
## Step 3: Evaluate the third term
The third term $i^{45}$ can be evaluated by rewriting the exponent as a sum of exponents of 4. Since 45 is equal to $4 \cdot 11 + 1$, $i^{45} = (i^4)^{11} \cdot i = 1^{11} \cdot i = i$.
## Step 4: Add the three terms together
The sum of the three terms is $i + i^{-25} + i^{45} = i + i^{-1} + i"
3,4,"If $2^8=4^x$, then $2^8=2^{2x}$, which implies that $8=2x$, so that $x=4$."
4,402,"The expression $\frac{2\left( 1-x\right)\left( 1+x\right)}{1-x^2}$ simplifies to $\boxed{2}$.
###
Problem Goal:
What"
5,"x \in [-2,7]","The solutions to the quadratic equation $x^2 - 5x - 4 = 0$ are $x = -1$ and $x = 4$. Since $-1$ is less than $-2$ and $7$ is greater than $7$, the solution set is $\boxed{[-2, 7]}$.
The final answer is: $\boxed{[-2, 7]}$
Note: The final answer is already provided in the solution, so there is no need to change it. The format is followed exactly as specified in the prompt."
6,-\frac{1,"The answer is $\boxed{-\frac{1}{8}}$.
Note: The answer is negative."
7,\frac{x+2,"The inverse of $h(x)$ is $\boxed{\frac{y+2}{7}}$.
In this problem, we are asked to find the inverse of a function $h(x)$. This means that we need to find a function $h^{-1}(x)$ that satisfies $h(h^{-1}(x))=x$. In other words, we need to find a function $h^{-1}(x)$ that undoes the function $h(x)$. To find $h^{-1}(x)$, we can use the following procedure:
1. Evaluate $h(x)$.
2. Write $h(x)=y$.
3. Solve for $x$.
4. Write the inverse function $h^{-1}(x)$.
Using this procedure, we can find the inverse of $h(x)$. First, we evaluate $h(x)$ as $h(x)=f(g(x))=f(x-1)=7(x-1)+5=7x-2$. Then we write $h(x)=y$ and solve for $x$ to get $x=\frac{y+2}{7}$. Therefore, the inverse function is $h^{-1}(x)=\frac{y+2}{7}$."
8,10,"The distance from the origin to the point $(-8,6)$ is $\boxed{10}$ units.
Note: The next step is a possible solution to the problem. The solution may be correct or incorrect. It is always a good idea to check the solution to make sure it is correct before presenting it as the final answer."
9,8,"The average of the numbers 2 and 4 is $\frac
The average of the numbers 5 and 7 is $\frac
The average of the numbers 6 and 8 is $\frac
The average of the numbers 9 and 11 is $\frac
The average of the numbers 1 and 3 is $\frac
The average of the numbers 2 and 4 is $\frac
The average of the numbers 5 and 7 is $\frac
The average of the numbers 6 and 8 is $\frac
The average of the numbers 9 and 11 is $\frac
The average of the numbers 2 and 4 is $\frac
The average of the numbers 5 and 7 is $\frac
The average of the numbers 6 and 8 is $\frac
The average of the numbers 9 and 11 is $\frac
The average of the numbers 2 and 4 is $\frac
The average of the numbers 5 and 7 is $\frac
The average of the numbers 6 and 8 is $\frac
The average of the numbers 9 and 11 is $\frac
The average of the numbers 2 and 4 is $\frac
The average of the numbers 5 and 7 is $\frac
The average of the numbers 6 and 8 is $\frac
The average of the numbers 9 and 11 is $\frac
The average of the numbers 2 and 4 is $\frac
The average of the numbers 5 and 7 is $\frac
The average of the numbers"
10,-25,The value of f(9
11,8,"Try another approach. Given that $A\sqrt{B}$ must equal $9C$, we can try to factor 9 out of the expression $\frac{A\sqrt{B}}{C}$. This is possible if we can factor out a 9 from the denominator. Since $C$ is a positive integer, $\frac{1}{C}$ is a positive rational number. Since $A$ and $C$ have no common prime factors, $C$ is an odd integer. Since $\frac{A\sqrt{B}}{C}$ is a positive rational number, $\sqrt{B}$ must be an odd integer.
## Step 18: Identify that B is 9
Since $B$ is a positive integer, $B^{2}$ is a perfect square factor of $B$. Since $B$ has no perfect-square factors other than 1, $B^{2}$ must equal 1. Thus, $B$ is 9.
## Step 19: Solve for C
Since $\frac{A\sqrt{B}}{C} = \frac{9}{2\sqrt{3}}$, $\sqrt{B}$ must equal $\frac{9}{2\sqrt{3}} \cdot C$. Thus, $C = \frac{\sqrt{B}}{\frac{9}{2\sqrt{3}}}$.
## Step 20: Solve for C
$C = \frac{\sqrt{B}}{\frac{9}{2\sqrt{3}}} = \frac{\sqrt{9}}{\frac{9}{2\sqrt{3}}} = \frac{3}{\frac{9}{2\sqrt{3}}} = \frac{3}{\frac{9}{2\sqrt{3}}} \cdot \frac{2\sqrt{3}}{2\sqrt{3}} = \frac{6\sqrt{3}}{9} = \frac{2\sqrt{3}}{3}$.
## Step 21: Solve for A
Since $B$ is 9, $\sqrt{B}$ is 3. Since $\frac{A\sqrt{B}}{C} = \frac{9}{2\sqrt{3}}$, $A\sqrt{B}$ must equal $9C$. Thus, $A\sqrt{B} = 9C$ implies that $A\sqrt{B} = 9C$ implies that $A = \frac{9C}{\sqrt{B}}$.
## Step 22: Solve for A
$A = \frac{9C}{\sqrt{B}} = \frac{9 \cdot \frac{2\sqrt{3}}{3}}{3} = \frac{9 \cdot \frac{2\sqrt{3}}{3}}{3} = \frac{9 \cdot 2\sqrt{3}}{9} = \frac{18\sqrt{3}}{9} = \frac{6\sqrt{3}}{3} = 2\sqrt{3}$.
## Step 23: Solve for A
$A = 2\sqrt{3}$.
## Step 24: Solve for A
$A = 2\sqrt{3}$.
## Step 25: Solve for A
$A = 2\sqrt{3}$.
## Step 26: Solve for A
Since $\frac{A\sqrt{B}}{C} = \frac{9}{2\sqrt{3}}$, $\sqrt{B}$ must equal $\frac{9}{2\sqrt{3}} \cdot C$. Thus, $C = \frac{\sqrt{B}}{\frac{9}{2\sqrt{3}}}$.
## Step 27: Solve for C
$C = \frac{\sqrt{B}}{\frac{9}{2\sqrt{3}}} = \frac{\sqrt{9}}{\frac{9}{2\sqrt{3}}} = \frac{3}{\frac{9}{2\sqrt{3}}} = \frac{3}{\frac{9}{2\sqrt{3}}} \cdot \frac{2\sqrt{3}}{2\sqrt{3}} = \frac{6\sqrt{3}}{9} = \frac{2\sqrt{3}}{3}$.
## Step 28: Solve for C
Since $A$ is $2\sqrt{3}$, $\frac{A\sqrt{B}}{C} = \frac{9}{2\sqrt{3}}$ implies that $\frac{2\sqrt{3}\sqrt{9}}{C} = \frac{9}{2\sqrt{3}}$.
## Step 29: Solve for C
$\frac{2\sqrt{3}\"
12,187.5,"The number of calories in a 12 fluid ounce bottle is 12 times 125, which is $\boxed{187.5}$.
Note that there is no possible next step for this problem as the solution is already found.
If you get stuck, check the work. Recheck your steps to ensure that you have not made a calculation error. In some cases, a simple error in a calculation can cause a big problem. If you have not found a possible next step and the current state does not solve the problem, consider the following suggestions:
\begin{enumerate}
\item Check your work: Look for any calculation errors.
\item Look for a possible simplification: See if the current state can be simplified or re-expressed.
\item Check for any assumptions: Verify that you have not assumed anything that is not given in the problem.
\item Look for an error in the problem statement: Double-check the problem statement for any errors.
\item Consider a different approach: Think about the problem from a different angle or perspective.
\item Consider a different current state: Re-evaluate your current state and look for a possible next step.
\end{enumerate}
If you have tried the above suggestions and you still cannot find a possible next step, consider consulting a classmate, teacher, or tutor. They may be able to help you find a possible next step."
13,\$40,"Mike will be offered $\$21.25$ in 12 more years.
The stamp's offer price doubles every 3 years. Thus, in 3 more years, Mike will be offered $\$21.25\cdot2= 42.50$.
The final answer is: $\boxed{42.50}$"
14,5,"If $17^6-17^5=17^x\cdot16$ then $17^5\cdot17=17^x\cdot16$. If $17^5=16$ then $17=1/16$ and $17^x=1$. This implies that $x=0$ and $17^x=17^0=1$. Thus, $x=\boxed0$."
15,8,The new pressure will be approximately $\boxed{27.11}$ kPa.
16,3125,a3⋅a2= a3⋅a2= a3⋅a2= a3⋅a2= a3⋅a2= a3⋅a2= a3⋅a2= a3⋅a2= a3⋅a2= a3⋅a2= a3⋅a2= a3⋅a2= a3⋅a2= a3⋅a2= a3⋅a2= a3⋅a2= a3⋅a2= a3⋅a2= a3⋅a2= a3⋅a2= a3⋅a2= a3⋅a2= a3⋅a2= a3⋅a2= a3⋅a2= a3⋅a2= a3⋅a2= a3⋅a2= a3⋅a2= a3⋅a2= a3⋅a2= a3⋅a2= a3⋅a2= a3⋅a2= a3⋅a2= a3⋅a2= a3⋅a2= a3⋅a2= a3<61>
17,"[0,\infty)","The range of the function is $[0, 4]$.
The range of the function is $[0, 4]$.
The range of the function is $[0, 4]$.
The range of the function is $[0, 4]$.
The range of the function is $[0, 4]$.
The range of the function is $[0, 4]$.
The range of the function is $[0, 4]$.
The range of the function is $[0, 4]$.
The range of the function is $[0, 4]$.
The range of the function is $[0, 4]$.
The range of the function is $[0, 4]$.
The range of the function is $[0, 4]$.
The range of the function is $[0, 4]$.
The range of the function is $[0, 4]$.
The range of the function is $[0, 4]$.
The range of the function is $[0, 4]$.
The range of the function is $[0, 4]$.
The range of the function is $[0, 4]$.
The range of the function is $[0, 4]$.
The range of the function is $[0, 4]$.
The range of the function is $[0, 4]$.
The range of the function is $[0, 4]$.
The range of the function is $[0, 4]$.
The range of the function is $[0, 4]$.
The range of the function is $[0, 4]$.
The range of the function is $[0, 4]$.
The range of the function is $[0, 4]$.
The range of the function is $[0, 4]$.
The range of the function is $[0, 4]$.
The range of the function is $[0, 4]$.
The range of the function is $[0, 4]$.
The range of the function is $[0, 4]$.
The range of the function is $[0, 4]$.
The range of the function is $[0, 4]$.
The range of the function is $[0, 4]$.
The range of the function is $[0, 4]$.
The range of the function is $[0, 4]$.
The range of the function is $[0, 4]$.
The range of the function is $[0, 4]$.
The range of the function is $[0, 4]$.
The range of the function is $[0, 4]$.
The range of the function is $[0, 4]$.
The range of the function is $[0, 4]$.
The range of the function is $[0, 4]$.
The range of the function is $[0, 4]$.
The range of the function is $[0, 4]$.
The range of the function is $[0, 4]$.
The range of the function is $[0, 4]$.
The range of the function is $[0, 4]$.
The range of the function is $[0, 4]$.
The range of the function is $[0, 4]$.
The range of the function is $[0, 4]$.
The range of the function is $[0, 4]$.
The range of the function is $[0, 4]$.
The range of the function is $[0, 4]$.
The range of the function is $[0, 4]$.
The range of the function is $[0, 4]$.
The range of the function is $[0, 4]$.
The range of the function is $[0"
18,.5,"The solutions to the equation are $x = 3$ and $x = \frac{1}{2}$. The sum of the possible values of $x$ is $\boxed(3+\frac{1}{2})$.
###"
19,5,"The difference between 36 and 26 is $\boxed{10}$. (Not needed, but a good check)"
20,105,"The sum of the elements of each of the 15 two-element subsets of $\{1,2,3,4,5,6\}$ is $\frac{15\cdot21}{2}=157.5$. \(\boxed{157.5}\)
Note the following things:
* The current state must be expressed as an equation.
* The next step must be a calculation or an expression of a calculation.
* The next step must be a single step to reach the goal.
* The goal is not always a numerical value. The goal can be an expression.
* If the current state is not a numerical value, the next step must be an expression."
21,2,"The value of $n$ is $\boxed{2}$.
###"
22,"(-\infty,-8)\cup (8,\infty)",The product of $(m-8)(m+8)$ is $(m^2-64)$. The solution to $m^2-64>0$ is $m<-8$ or $m>8$.
23,0,"We can check the solution by substituting $x = -1/2$ into the equation $2x^2 + 3x + 2 = 0$ to get $2(-1/2)^2 + 3(-1/2) + 2 = 0$, which implies that $(1/2) + (-3/2) + 2 = 0$, which implies that $-1/2 + 2 = 0$, which implies that $3/2 = 0$, which is a false statement. Therefore, we need to find the other solution to the equation $2x^2 + 3x + 2 = 0$. We can use the quadratic formula $x = {-b\pm \sqrt{b^2 - 4ac} 4a"
24,2,"The values that $x$ can take are $8$ and $-8$. Therefore, the sum of all values of $x$ such that $|x-1| = 7$ is $\boxed{0}$.
The values that $x$ can take are $8$ and $-8$. Therefore, the sum of all values of $x$ such that $|x-1| = 7$ is $\boxed{0}$.
The values that $x$ can take are $8$ and $-8$. Therefore, the sum of all values of $x$ such that $|x-1| = 7$ is $\boxed{0}$.
The values that $x$ can take are $8$ and $-8$. Therefore, the sum of all values of $x$ such that $|x-1| = 7$ is $\boxed{0}$.
The values that $x$ can take are $8$ and $-8$. Therefore, the sum of all values of $x$ such that $|x-1| = 7$ is $\boxed{0}$.
The values that $x$ can take are $8$ and $-8$. Therefore, the sum of all values of $x$ such that $|x-1| = 7$ is $\boxed{0}$.
The values that $x$ can take are $8$ and $-8$. Therefore, the sum of all values of $x$ such that $|x-1| = 7$ is $\boxed{0}$.
The values that $x$ can take are $8$ and $-8$. Therefore, the sum of all values of $x$ such that $|x-1| = 7$ is $\boxed{0}$.
The values that $x$ can take are $8$ and $-8$. Therefore, the sum of all values of $x$ such that $|x-1| = 7$ is $\boxed{0}$.
The values that $x$ can take are $8$ and $-8$. Therefore, the sum of all values of $x$ such that $|x-1| = 7$ is $\boxed{0}$.
The values that $x$ can take are $8$ and $-8$. Therefore, the sum of all values of $x$ such that $|x-1| = 7$ is $\boxed{0}$.
The values that $x$ can take are $8$ and $-8$. Therefore, the sum of all values of $x$ such that $|x-1| = 7$ is $\boxed{0}$.
The values that $x$ can take are $8$ and $-8$. Therefore, the sum of all values of $x$ such that $|x-1| = 7$ is $\boxed{0}$.
The values that $x$ can take are $8$ and $-8$. Therefore, the sum of all values of $x$ such that $|x-1| = 7$ is $\boxed{0}$.
The values that $x$ can take are $8$ and $-8$. Therefore, the sum of all values of $x$ such that $|x-1| = 7$ is $\boxed{0}$.
The values that $x$ can take are $8$ and $-8$. Therefore, the sum of all values of $x$ such that $|x-1| = 7$ is $\boxed{0}$.
The values that $x$ can take are $8$ and $-8$. Therefore, the sum of all values of $x$ such that $|x-1| = 7$ is $\boxed{0}$.
The values that $x$ can take are $8$ and $-8$. Therefore, the sum of all values of $x$ such that $|x-1"
25,\frac{1,"Solve for $w$ by multiplying both sides by $w$ and then multiplying both sides by 9 to get rid of the denominators and fractions. Then solve for $w$ to get $w=\frac{9\cdot42}{1\cdot3}=\frac{378}{3}=126$.
###
Problem Goal:
What is the value of $y$ in the equation $y$ $y$ $y$ $y$ $y$ $y$ $y$ $y$ $y$ $y$ $y$ $y$ $y$ $y$ $y$ $y$ $y$ $y$ $y$ $y$ $y$ $y$ $y$ $y$ $y$ $y$ $y$ $y$ $y$ $y$ $y$ $y$ $y$ $y$ $y$ $y$ $y$ $y$ $y$ $y$ $y$ $y$ $y$ $y$ $y$ $y$ $y$ $y$ $y$ $y$ $y$ $y$ $y$ $y$ $y$ $y$ $y$ $y$ $y$ $y$ $y$ $y$ $y$ $y$ $y$ $y$ $y$ $y$ $y$ $y$ $y$ $y$ $y$ $y$ $y"
26,6+9i,"We can evaluate $(1+2i)^3$ by expanding it in the form of $(a+b)^3$.
\begin{align*} (1+2i)^3 &amp;= (1+2i)^3 \\ &amp;= 1^3 + 3(1)^2(2i) + 3(1)(2i)^2 + (2i)^3 \\ &amp;= 1 + 6i -12 - 8i \\ &amp;= -11 - 2i \end{align*}.
Now we have $(1-2i)^3(-11-2i)$.
We can simplify this expression further to find the value of $\left( 1 - 2i \right)^3 \left( 1 + 2i \right)^{3-3i}$.
We have
\begin{align*} \left( 1 - 2i \right)^3 \left( 1 + 2i \right)^{3-3i} &amp;= \left( -11 - 2i \right) \left( 1 + 2i \right)^{3-3i} \\ &amp;= (-11 - 2i)(1 + 2i)^{3-3i} \\ &amp;= (-11 - 2i)(1+2i)^3 \\ &amp;= (-11 - 2i)(-11-2i) \\ &amp;= (121+22i+4i^2) \\ &amp;= 121+22i-4 \\ &amp;= 117+22i. \end{align*},
Note: $i^2=-1$.
The final answer is $\boxed{117+22i}$.
Note that in this problem, the property $(ab)^n = a^n b^n$ was used with $a=1+2i$, $b=1-2i$ and $n=3-3i$. It was also necessary to simplify the expression $\left( 1 - 2i \right)^3 \left( 1 + 2i \right)^{3-3i}$ by expanding the expression $\left( 1 + 2i \right)^3$ in the form of $(a+b)^3$. This problem also required the evaluation of $i^2$.
### Problem Goal:
Evaluate $(x+2i)5-5i$.
\begin{align*} \left( x+2i \right)^5 \left( x+2i \right)^{-5i} &amp;= \left( x+2i \right)^5 \left( x+2i \right)^{5-5i} \\ &amp;= \left( x+2i \right)^5 \left( x+2i \right)^{5-5i} \\ &amp;= \left( x+2i \right)^5 \left( x+2i \right)^{5-5i} \end{align*}, where we applied the property $(ab)^n = a^n b^n$ for $a=x+2i$, $b=1$ and $n=5-5i$.
We can now substitute the value of $(x+2i)^5$ in the above equation to find the value of $\left( x+2i \right)^5 \left( x+2i \right)^{5-5i}$.
We have
\begin{align*} \left( x+2i \right)^5 \left( x+2i \right)^{5-5i} &amp;= \left( x+2i \right)^5 \left( x+2"
27,2,"The simplified numerical value of $\frac{a+11b}{a-b}$ is $\boxed{2}$.
In the previous example, we have a system of linear equations in the form of $\frac{a+11b}{a-b}$ and $\frac{4a+3b}{a-2b}$. The current state is $\frac{4a+3b}{a-2b}$ = 5. We will now solve this system of linear equations.
First, we will solve $\frac{4a+3b}{a-2b}$ = 5 to $4"
28,20,"First, multiply the two complex numbers, \begin{align*} (2-2i)(5+5i)&=2\cdot 5+2\cdot 5i-2i\cdot 5-2i\cdot 5i\\ &\qquad\qquad\qquad\qquad\qquad\qquad\qquad\qquad\qquad\qquad\qquad\qquad\qquad\qquad\qquad\qquad\qquad\qquad\qquad\qquad\qquad\qquad\qquad\qquad\qquad\qquad\qquad\qquad\qquad\qquad\qquad\qquad\qquad\qquad\qquad\qquad\qquad\qquad\qquad\qquad\qquad\qquad\qquad\qquad\qquad\qquad\qquad\qquad\qquad\qquad\qquad\qquad\qquad\qquad\qquad\qquad\qquad\qquad\qquad\qquad\qquad\qquad\qquad\qquad\qquad\qquad\qquad\qquad\qquad\qquad\qquad\qquad\qquad\qquad\qquad\qquad\qquad\qquad\qquad\qquad\qquad\qquad\qquad\qquad\qquad\qquad\qquad\qquad\qquad\qquad\qquad\qquad\qquad\qquad\qquad\qquad\qquad\qquad\qquad\qquad\qquad\qquad\qquad\qquad\qquad\qquad\qquad\qquad\qquad\qquad\qquad\qquad\qquad\qquad\qquad\qquad\qquad\qquad\qquad\qquad\qquad\qquad\qquad\qquad\qquad\qquad\qquad\qquad\qquad\qquad\qquad\qquad\qquad\qquad\qquad\qquad\qquad\qquad\qquad\qquad\qquad\qquad\qquad\qquad\qquad\qquad\qquad\qquad\qquad\qquad\qquad\qquad\qquad\qquad\qquad\qquad\qquad\qquad\qquad\qquad\qquad\qquad\qquad\qquad\qquad\qquad\qquad\qquad\qquad\qquad\qquad\qquad\qquad\qquad\qquad\qquad\qquad\qquad\qquad\qquad\qquad\qquad\qquad\qquad\qquad\qquad\qquad\qquad\qquad\qquad\qquad\qquad\qquad\qquad\qquad\qquad\qquad\qquad\qquad\qquad\qquad\qquad\qquad\qquad\qquad\qquad\qquad\qquad\qquad\qquad\qquad\qquad\qquad\qquad\qquad\qquad\qquad\qquad\qquad\qquad\qquad\qquad\qquad\qquad\qquad\qquad\qquad\qquad\qquad\qquad\qquad\qquad\qquad\qquad\qquad\qquad\qquad\qquad\qquad\qquad\qquad\qquad\qquad\qquad\qquad\qquad\qquad\qquad\qquad\qquad\qquad\qquad\qquad\qquad\qquad\qquad\qquad\qquad\qquad\qquad\qquad"
29,7(x+3) (x-3),"The expression can be factored into the product of two binomials, $7(x - \sqrt{9})(x + \sqrt{9})$."
30,y^4-2y^3+7y^2+y-5,"The volume of a cone with a radius of $ 5$, and height of $ 8$ is $\frac{1}{3} \pi (5)^2 (8) = \frac{1}{3} \pi 25 (8) = \frac{200\pi}{3}$.
The final answer is $\frac{200\pi}{3}$.
###
Problem Goal:
A population of rabbits is growing at a rate of $ 10\%$"
31,0,"\begin{align*}
f(x)-f^{-1}(x)&=x^2-\frac{1}{x}\\
&=x^2-\frac{x}{x^2}\\
&=x^2-\frac{1}{x}\\
&=x^2-x^{-1}
\end{align*}
Note: The previous solution had an error. The solution was corrected in the above solution.
Note: The above solution is correct."
32,\frac{7,The car travels 70 miles.
33,5,"The expression $64\sqrt{8}$ is equivalent to $(4^3)(8^{1/2})$ which is equivalent to $(4^3)(8^{1/2})$ which is equivalent to $((4^3)(8^{1/2}))^{1/3}$ which is equivalent to $(8^{1/2})^{1/3}$ which is equivalent to $(8^{1/2\cdot(1/3)})$ which is equivalent to $(8^{1/6})$ which is equivalent to $(8^{1/6})$ which is equivalent to $(\sqrt[6]{8})^{1/2}$ which is equivalent to $(\sqrt[6]{8})^{1/2}$ which is equivalent to $(\sqrt[6]{8})^{1/2}$ which is equivalent to $\boxed{\frac{7}{4}}$.
Note: The final answer $\boxed{\frac{7}{4}}$ is obtained by solving the problem from the current state."
34,"\left(-\infty,-\frac 12\right)\cup \left(-\frac 12,\infty\right)","The domain of the function is all real numbers except $y = -\frac{1}{2}$.
The final answer is: $\boxed{(-\infty, -\frac{1}{2}) \cup (-\frac{1}{2}, \infty)}$"
35,\frac{2,"The probability that there are no real solutions to the equation $x(x+5)=-n$ for the $n$ I chose is the probability that $n$ is between 7 and 10 inclusive, which is $\boxed{\frac{4}{10}=\frac{2}{5}}$."
36,7,"Note that $f^{-1}(x)$ is the inverse of $f(x)$, so $f^{-1}(f^{-1}(3))=f^{-1}(7)=1$."
37,20,"The product of $(3-i)(6+2i)$ is $18+6i-6i-2i^{2}=18-2i^{2}=18+2=20$.
The final answer is $20$.
The final answer is \boxed{20}."
38,9,"The product of the squares of the solutions is $\boxed{(\frac{36 - 104}{4})=\frac{(-68)}{4}=\frac{-68}{4}=\frac{-34}{2}=-17}$.
The solutions to the quadratic equation are $x=\frac{-13\pm\sqrt{169-24\cdot2\cdot6}}{4}$.
The solutions to the quadratic equation are $x=\frac{-13\pm\sqrt{169-48}}{4}$.
The solutions to the quadratic equation are $x=\frac{-13\pm\sqrt{121}}{4}$.
The solutions to the quadratic equation are $x=\frac{-13\pm11}{4}$.
The solutions to the quadratic equation are $x=\frac{-13+11}{4}$.
The solutions to the quadratic equation are $x=\frac{-2}{4}$.
The solutions to the quadratic equation are $x=\frac{-2}{4}$.
The solutions to the quadratic equation are $x=-\frac{1}{2}$.
The solutions to the quadratic equation are $x=-\frac{1}{2}$.
The solutions to the quadratic equation are $x=-\frac{1}{2}$.
The solutions to the quadratic equation are $x=-\frac{1}{2}$.
The solutions to the quadratic equation are $x=-\frac{1}{2}$.
The solutions to the quadratic equation are $x=-\frac{1}{2}$.
The solutions to the quadratic equation are $x=-\frac{1}{2}$.
The solutions to the quadratic equation are $x=-\frac{1}{2"
39,4,"(16)^(1/3) is equal to 2.000000, and (2.000000)^(3) is equal to 4.000000, so the final answer is \boxed 4.
Note: You can only post one problem per time. Please rephrase the problem in a different way if you want to post another problem.
## Step 1: Understand the problem statement
The problem asks for the positive difference between $120\%$ of 30 and $130\%$ of 20.
## Step 2: Calculate the values
First, we need to calculate $120\%$ of 30. To do this, we multiply 30 by $120\%$, which is equivalent to $1.2$ times 30. This gives us $1.2 \times 30 = 36$.
## Step 3: Continue calculating the values
Next, we calculate $130\%$ of 20. To do this, we multiply 20 by $130\%$, which is equivalent to $1.3$ times 20. This gives us $1.3 \times 20 = 26$.
## Step 4: Find the positive difference
Now, we need to find the positive difference between 36 and 26. To do this, we subtract 26 from 36. This gives us $36 - 26 = 10$.
The final answer is: $\boxed{10}$"
40,6,"The absolute difference between 7 and 1 is $\boxed(6)$.
The absolute difference between 1 and 7 is $\boxed(6)$.
The absolute difference between $|m-n|$ and $6$ is $\boxed(6)$."
41,"\left(1,\frac{9","The height of the prism is $\boxed{12\text{ in}}$.
The height of the prism is 12 in.
###"
42,-2,"$
x = 2$ or $x = 3$
$
The roots are $2$ and $3$.
###
Problem Goal:
Solve $x^2 + 7x + 12 = 0$ by factoring.
$ x^2 + 7x + 12 = (x + 3)(x + 4) = 0$
$
$ x"
43,8,"The value of $y^2 - 3xy + 8$ for the pair $(x,y) = (4,5)$ is $\boxed{-27}$.
In summary, we solved the problem of calculating the value of the expression $y^2 - 3xy + 8$ for different pairs of coordinates $(x,y)$ by substituting the values of $x$ and $y$ into the expression and performing the necessary calculations. We obtained the value of $y^2 - 3xy + 8$ for the pair $(x,y) = (3,9)$ as $8$ and for the pair $(x,y) = (4,5)$ as $-27$."
44,20,"Substituting x=20 into $5x$ gives $5\cdot 20 = 100$, and substituting x=8 into $5x$ gives $5\cdot 8 = 40$. The difference between 100 and 40 is $\boxed{60}$."
45,24,"Find the value of $k$ when $j = 10$.
Since $j$ and $k$ are inversely proportional, we have that $j \cdot k = c$ where $c$ is a constant.
Since $j = 16$ when $k = 21$, we have that $16 \cdot 21 = c$, so $c = 336$.
Then $j \cdot k = 336$.
When $j = 10$ we have that $10 \cdot k = 336$.
Then $k = 336/10 = 33.6$.
Then the value of $j$ when $k = 14$ is $10 \cdot 14 = 140$.
Then the value of $k$ when $j = 12$ is $12 \cdot 25 = 300$.
Then the value of $j$ when $k = 14$ is $336/14 = 24$.
Then the value of $k$ when $j = 12$ is $336/12 = 28$.
Then the value of $j$ when $k = 14$ is $336/28 = 12$.
Then the value of $k$ when $j = 12$ is $336/12 = 28$.
Then the value of $j$ when $k = 14$ is $336/28 = 12$.
Then the value of $k$ when $j = 12$ is $336/28 = 12$.
Then the value of $j$ when $k = 14$ is $336/28 = 12$.
Then the value of $k$ when $j = 12$ is $336/28 = 12$.
Then the value of"
46,5,"The sum of all possible values of $x$ is $\boxed{5}$
This example shows how to take a step from the current state to a solution of a problem. We analyze the problem goal, the current state, and the difference between the problem goal and the current state. We also check if the current state is a solution to the problem, and if not, we propose a possible next step to solve the problem."
47,-7,"$\frac{2x-1}{2x+2}$ = $\frac{x-3}{x-1}$
for $x \neq -1$ and $x \neq 1$.
$2x - 1 = (2x + 2) (\frac{x-3}{x-1})$
$2x - 1 = (2x + 2) (\frac{x^2 - 3x -2x + 6}{x^2 -1}$
$2x - 1 = (2x + 2) (\frac{x^2 -5x +6}{x^2 -1}$
$2x - 1 = (2x + 2) (\frac{x^2 -5x +6}{x-1} (x+1)$
$2x - 1 = (2x + 2) (\frac{x(x^2 -5x +6)}{x^2 -1}$
$2x - 1 = (2x + 2) (\frac{x^3 -5x^2 +6x}{x^2 -1}$
$2x - 1 = (2x + 2) (\frac{x(x^2 -5x +6)}{(x+1)(x-1)}$
$2x - 1 = (2x + 2) (\frac{x(x^2 -5x +6)}{(x+1)(x-1)}$
$2x - 1 = (2x + 2) (\frac{x(x^2 -5x +6)}{(x+1)(x-1)}$
$2x - 1 = (2x + 2) (\frac{x(x-1)(x+5)}{(x+1)(x-1)}$
$2x - 1 = (2x + 2) (\frac{x(x+5)}{1}$
$2x - 1 = (2x + 2) (x+5)$
$2x - 1 = 2x^2 + 10x + 2x + 10$
$2x - 1 = 2x^2 + 12x + 10$
$2x^2 + 12x + 10 = 2x^2 -1$
$2x^2 + 12x + 10 = 2x^2 -1$
$2x^2 + 12x + 10 = 2x^2 -"
48,0.43,The ball hits the ground in $\boxed{0.37}$ seconds.
49,108,"The circumference of the first horse is 64/27=2.37 feet. The circumference of the second horse is 2 * π * 16=100.42 feet. The second horse must travel 64 feet. Since the second horse's circumference is 100.42 feet, the second horse must make $\frac{64}{100.42}$ = 0.6364 revolutions."
1 gt pred
2 0 2 The graph of $y=\frac{2}{x^2+x-6}$ has $\boxed{2}$ vertical asymptotes at x=2 and x=-3. The final answer is 2.
3 1 10 The difference between 36 and 26 is $\boxed{10}$. The final answer is: $\boxed{10}$
4 2 i The first term $i^5$ can be evaluated by rewriting the exponent as a sum of exponents of 4. Since 5 is equal to $4 \cdot 1 + 1$, $i^5 = (i^4)^1 \cdot i = 1^1 \cdot i = i$. ## Step 1: Evaluate the first term The first term $i^5$ can be evaluated by rewriting the exponent as a sum of exponents of 4. Since 5 is equal to $4 \cdot 1 + 1$, $i^5 = (i^4)^1 \cdot i = 1^1 \cdot i = i$. ## Step 2: Evaluate the second term The second term $i^{-25}$ can be evaluated by rewriting the exponent as a sum of exponents of 4. Since 25 is equal to $4 \cdot 6 + 1$, $i^{-25} = (i^4)^{-6} \cdot i^{-1} = 1^{-6} \cdot i^{-1} = 1^{-6} \cdot (i^4)^{-1} \cdot i = 1^{-6} \cdot 1^{-1} \cdot i = 1^{-6} \cdot i = i^{-1}$. ## Step 3: Evaluate the third term The third term $i^{45}$ can be evaluated by rewriting the exponent as a sum of exponents of 4. Since 45 is equal to $4 \cdot 11 + 1$, $i^{45} = (i^4)^{11} \cdot i = 1^{11} \cdot i = i$. ## Step 4: Add the three terms together The sum of the three terms is $i + i^{-25} + i^{45} = i + i^{-1} + i
5 3 4 If $2^8=4^x$, then $2^8=2^{2x}$, which implies that $8=2x$, so that $x=4$.
6 4 402 The expression $\frac{2\left( 1-x\right)\left( 1+x\right)}{1-x^2}$ simplifies to $\boxed{2}$. ### Problem Goal: What
7 5 x \in [-2,7] The solutions to the quadratic equation $x^2 - 5x - 4 = 0$ are $x = -1$ and $x = 4$. Since $-1$ is less than $-2$ and $7$ is greater than $7$, the solution set is $\boxed{[-2, 7]}$. The final answer is: $\boxed{[-2, 7]}$ Note: The final answer is already provided in the solution, so there is no need to change it. The format is followed exactly as specified in the prompt.
8 6 -\frac{1 The answer is $\boxed{-\frac{1}{8}}$. Note: The answer is negative.
9 7 \frac{x+2 The inverse of $h(x)$ is $\boxed{\frac{y+2}{7}}$. In this problem, we are asked to find the inverse of a function $h(x)$. This means that we need to find a function $h^{-1}(x)$ that satisfies $h(h^{-1}(x))=x$. In other words, we need to find a function $h^{-1}(x)$ that undoes the function $h(x)$. To find $h^{-1}(x)$, we can use the following procedure: 1. Evaluate $h(x)$. 2. Write $h(x)=y$. 3. Solve for $x$. 4. Write the inverse function $h^{-1}(x)$. Using this procedure, we can find the inverse of $h(x)$. First, we evaluate $h(x)$ as $h(x)=f(g(x))=f(x-1)=7(x-1)+5=7x-2$. Then we write $h(x)=y$ and solve for $x$ to get $x=\frac{y+2}{7}$. Therefore, the inverse function is $h^{-1}(x)=\frac{y+2}{7}$.
10 8 10 The distance from the origin to the point $(-8,6)$ is $\boxed{10}$ units. Note: The next step is a possible solution to the problem. The solution may be correct or incorrect. It is always a good idea to check the solution to make sure it is correct before presenting it as the final answer.
11 9 8 The average of the numbers 2 and 4 is $\frac The average of the numbers 5 and 7 is $\frac The average of the numbers 6 and 8 is $\frac The average of the numbers 9 and 11 is $\frac The average of the numbers 1 and 3 is $\frac The average of the numbers 2 and 4 is $\frac The average of the numbers 5 and 7 is $\frac The average of the numbers 6 and 8 is $\frac The average of the numbers 9 and 11 is $\frac The average of the numbers 2 and 4 is $\frac The average of the numbers 5 and 7 is $\frac The average of the numbers 6 and 8 is $\frac The average of the numbers 9 and 11 is $\frac The average of the numbers 2 and 4 is $\frac The average of the numbers 5 and 7 is $\frac The average of the numbers 6 and 8 is $\frac The average of the numbers 9 and 11 is $\frac The average of the numbers 2 and 4 is $\frac The average of the numbers 5 and 7 is $\frac The average of the numbers 6 and 8 is $\frac The average of the numbers 9 and 11 is $\frac The average of the numbers 2 and 4 is $\frac The average of the numbers 5 and 7 is $\frac The average of the numbers
12 10 -25 The value of f(9
13 11 8 Try another approach. Given that $A\sqrt{B}$ must equal $9C$, we can try to factor 9 out of the expression $\frac{A\sqrt{B}}{C}$. This is possible if we can factor out a 9 from the denominator. Since $C$ is a positive integer, $\frac{1}{C}$ is a positive rational number. Since $A$ and $C$ have no common prime factors, $C$ is an odd integer. Since $\frac{A\sqrt{B}}{C}$ is a positive rational number, $\sqrt{B}$ must be an odd integer. ## Step 18: Identify that B is 9 Since $B$ is a positive integer, $B^{2}$ is a perfect square factor of $B$. Since $B$ has no perfect-square factors other than 1, $B^{2}$ must equal 1. Thus, $B$ is 9. ## Step 19: Solve for C Since $\frac{A\sqrt{B}}{C} = \frac{9}{2\sqrt{3}}$, $\sqrt{B}$ must equal $\frac{9}{2\sqrt{3}} \cdot C$. Thus, $C = \frac{\sqrt{B}}{\frac{9}{2\sqrt{3}}}$. ## Step 20: Solve for C $C = \frac{\sqrt{B}}{\frac{9}{2\sqrt{3}}} = \frac{\sqrt{9}}{\frac{9}{2\sqrt{3}}} = \frac{3}{\frac{9}{2\sqrt{3}}} = \frac{3}{\frac{9}{2\sqrt{3}}} \cdot \frac{2\sqrt{3}}{2\sqrt{3}} = \frac{6\sqrt{3}}{9} = \frac{2\sqrt{3}}{3}$. ## Step 21: Solve for A Since $B$ is 9, $\sqrt{B}$ is 3. Since $\frac{A\sqrt{B}}{C} = \frac{9}{2\sqrt{3}}$, $A\sqrt{B}$ must equal $9C$. Thus, $A\sqrt{B} = 9C$ implies that $A\sqrt{B} = 9C$ implies that $A = \frac{9C}{\sqrt{B}}$. ## Step 22: Solve for A $A = \frac{9C}{\sqrt{B}} = \frac{9 \cdot \frac{2\sqrt{3}}{3}}{3} = \frac{9 \cdot \frac{2\sqrt{3}}{3}}{3} = \frac{9 \cdot 2\sqrt{3}}{9} = \frac{18\sqrt{3}}{9} = \frac{6\sqrt{3}}{3} = 2\sqrt{3}$. ## Step 23: Solve for A $A = 2\sqrt{3}$. ## Step 24: Solve for A $A = 2\sqrt{3}$. ## Step 25: Solve for A $A = 2\sqrt{3}$. ## Step 26: Solve for A Since $\frac{A\sqrt{B}}{C} = \frac{9}{2\sqrt{3}}$, $\sqrt{B}$ must equal $\frac{9}{2\sqrt{3}} \cdot C$. Thus, $C = \frac{\sqrt{B}}{\frac{9}{2\sqrt{3}}}$. ## Step 27: Solve for C $C = \frac{\sqrt{B}}{\frac{9}{2\sqrt{3}}} = \frac{\sqrt{9}}{\frac{9}{2\sqrt{3}}} = \frac{3}{\frac{9}{2\sqrt{3}}} = \frac{3}{\frac{9}{2\sqrt{3}}} \cdot \frac{2\sqrt{3}}{2\sqrt{3}} = \frac{6\sqrt{3}}{9} = \frac{2\sqrt{3}}{3}$. ## Step 28: Solve for C Since $A$ is $2\sqrt{3}$, $\frac{A\sqrt{B}}{C} = \frac{9}{2\sqrt{3}}$ implies that $\frac{2\sqrt{3}\sqrt{9}}{C} = \frac{9}{2\sqrt{3}}$. ## Step 29: Solve for C $\frac{2\sqrt{3}\
14 12 187.5 The number of calories in a 12 fluid ounce bottle is 12 times 125, which is $\boxed{187.5}$. Note that there is no possible next step for this problem as the solution is already found. If you get stuck, check the work. Recheck your steps to ensure that you have not made a calculation error. In some cases, a simple error in a calculation can cause a big problem. If you have not found a possible next step and the current state does not solve the problem, consider the following suggestions: \begin{enumerate} \item Check your work: Look for any calculation errors. \item Look for a possible simplification: See if the current state can be simplified or re-expressed. \item Check for any assumptions: Verify that you have not assumed anything that is not given in the problem. \item Look for an error in the problem statement: Double-check the problem statement for any errors. \item Consider a different approach: Think about the problem from a different angle or perspective. \item Consider a different current state: Re-evaluate your current state and look for a possible next step. \end{enumerate} If you have tried the above suggestions and you still cannot find a possible next step, consider consulting a classmate, teacher, or tutor. They may be able to help you find a possible next step.
15 13 \$40 Mike will be offered $\$21.25$ in 12 more years. The stamp's offer price doubles every 3 years. Thus, in 3 more years, Mike will be offered $\$21.25\cdot2= 42.50$. The final answer is: $\boxed{42.50}$
16 14 5 If $17^6-17^5=17^x\cdot16$ then $17^5\cdot17=17^x\cdot16$. If $17^5=16$ then $17=1/16$ and $17^x=1$. This implies that $x=0$ and $17^x=17^0=1$. Thus, $x=\boxed0$.
17 15 8 The new pressure will be approximately $\boxed{27.11}$ kPa.
18 16 3125 a3⋅a2= a3⋅a2= a3⋅a2= a3⋅a2= a3⋅a2= a3⋅a2= a3⋅a2= a3⋅a2= a3⋅a2= a3⋅a2= a3⋅a2= a3⋅a2= a3⋅a2= a3⋅a2= a3⋅a2= a3⋅a2= a3⋅a2= a3⋅a2= a3⋅a2= a3⋅a2= a3⋅a2= a3⋅a2= a3⋅a2= a3⋅a2= a3⋅a2= a3⋅a2= a3⋅a2= a3⋅a2= a3⋅a2= a3⋅a2= a3⋅a2= a3⋅a2= a3⋅a2= a3⋅a2= a3⋅a2= a3⋅a2= a3⋅a2= a3⋅a2= a3�
19 17 [0,\infty) The range of the function is $[0, 4]$. The range of the function is $[0, 4]$. The range of the function is $[0, 4]$. The range of the function is $[0, 4]$. The range of the function is $[0, 4]$. The range of the function is $[0, 4]$. The range of the function is $[0, 4]$. The range of the function is $[0, 4]$. The range of the function is $[0, 4]$. The range of the function is $[0, 4]$. The range of the function is $[0, 4]$. The range of the function is $[0, 4]$. The range of the function is $[0, 4]$. The range of the function is $[0, 4]$. The range of the function is $[0, 4]$. The range of the function is $[0, 4]$. The range of the function is $[0, 4]$. The range of the function is $[0, 4]$. The range of the function is $[0, 4]$. The range of the function is $[0, 4]$. The range of the function is $[0, 4]$. The range of the function is $[0, 4]$. The range of the function is $[0, 4]$. The range of the function is $[0, 4]$. The range of the function is $[0, 4]$. The range of the function is $[0, 4]$. The range of the function is $[0, 4]$. The range of the function is $[0, 4]$. The range of the function is $[0, 4]$. The range of the function is $[0, 4]$. The range of the function is $[0, 4]$. The range of the function is $[0, 4]$. The range of the function is $[0, 4]$. The range of the function is $[0, 4]$. The range of the function is $[0, 4]$. The range of the function is $[0, 4]$. The range of the function is $[0, 4]$. The range of the function is $[0, 4]$. The range of the function is $[0, 4]$. The range of the function is $[0, 4]$. The range of the function is $[0, 4]$. The range of the function is $[0, 4]$. The range of the function is $[0, 4]$. The range of the function is $[0, 4]$. The range of the function is $[0, 4]$. The range of the function is $[0, 4]$. The range of the function is $[0, 4]$. The range of the function is $[0, 4]$. The range of the function is $[0, 4]$. The range of the function is $[0, 4]$. The range of the function is $[0, 4]$. The range of the function is $[0, 4]$. The range of the function is $[0, 4]$. The range of the function is $[0, 4]$. The range of the function is $[0, 4]$. The range of the function is $[0, 4]$. The range of the function is $[0, 4]$. The range of the function is $[0, 4]$. The range of the function is $[0
20 18 .5 The solutions to the equation are $x = 3$ and $x = \frac{1}{2}$. The sum of the possible values of $x$ is $\boxed(3+\frac{1}{2})$. ###
21 19 5 The difference between 36 and 26 is $\boxed{10}$. (Not needed, but a good check)
22 20 105 The sum of the elements of each of the 15 two-element subsets of $\{1,2,3,4,5,6\}$ is $\frac{15\cdot21}{2}=157.5$. \(\boxed{157.5}\) Note the following things: * The current state must be expressed as an equation. * The next step must be a calculation or an expression of a calculation. * The next step must be a single step to reach the goal. * The goal is not always a numerical value. The goal can be an expression. * If the current state is not a numerical value, the next step must be an expression.
23 21 2 The value of $n$ is $\boxed{2}$. ###
24 22 (-\infty,-8)\cup (8,\infty) The product of $(m-8)(m+8)$ is $(m^2-64)$. The solution to $m^2-64>0$ is $m<-8$ or $m>8$.
25 23 0 We can check the solution by substituting $x = -1/2$ into the equation $2x^2 + 3x + 2 = 0$ to get $2(-1/2)^2 + 3(-1/2) + 2 = 0$, which implies that $(1/2) + (-3/2) + 2 = 0$, which implies that $-1/2 + 2 = 0$, which implies that $3/2 = 0$, which is a false statement. Therefore, we need to find the other solution to the equation $2x^2 + 3x + 2 = 0$. We can use the quadratic formula $x = {-b\pm \sqrt{b^2 - 4ac} 4a
26 24 2 The values that $x$ can take are $8$ and $-8$. Therefore, the sum of all values of $x$ such that $|x-1| = 7$ is $\boxed{0}$. The values that $x$ can take are $8$ and $-8$. Therefore, the sum of all values of $x$ such that $|x-1| = 7$ is $\boxed{0}$. The values that $x$ can take are $8$ and $-8$. Therefore, the sum of all values of $x$ such that $|x-1| = 7$ is $\boxed{0}$. The values that $x$ can take are $8$ and $-8$. Therefore, the sum of all values of $x$ such that $|x-1| = 7$ is $\boxed{0}$. The values that $x$ can take are $8$ and $-8$. Therefore, the sum of all values of $x$ such that $|x-1| = 7$ is $\boxed{0}$. The values that $x$ can take are $8$ and $-8$. Therefore, the sum of all values of $x$ such that $|x-1| = 7$ is $\boxed{0}$. The values that $x$ can take are $8$ and $-8$. Therefore, the sum of all values of $x$ such that $|x-1| = 7$ is $\boxed{0}$. The values that $x$ can take are $8$ and $-8$. Therefore, the sum of all values of $x$ such that $|x-1| = 7$ is $\boxed{0}$. The values that $x$ can take are $8$ and $-8$. Therefore, the sum of all values of $x$ such that $|x-1| = 7$ is $\boxed{0}$. The values that $x$ can take are $8$ and $-8$. Therefore, the sum of all values of $x$ such that $|x-1| = 7$ is $\boxed{0}$. The values that $x$ can take are $8$ and $-8$. Therefore, the sum of all values of $x$ such that $|x-1| = 7$ is $\boxed{0}$. The values that $x$ can take are $8$ and $-8$. Therefore, the sum of all values of $x$ such that $|x-1| = 7$ is $\boxed{0}$. The values that $x$ can take are $8$ and $-8$. Therefore, the sum of all values of $x$ such that $|x-1| = 7$ is $\boxed{0}$. The values that $x$ can take are $8$ and $-8$. Therefore, the sum of all values of $x$ such that $|x-1| = 7$ is $\boxed{0}$. The values that $x$ can take are $8$ and $-8$. Therefore, the sum of all values of $x$ such that $|x-1| = 7$ is $\boxed{0}$. The values that $x$ can take are $8$ and $-8$. Therefore, the sum of all values of $x$ such that $|x-1| = 7$ is $\boxed{0}$. The values that $x$ can take are $8$ and $-8$. Therefore, the sum of all values of $x$ such that $|x-1| = 7$ is $\boxed{0}$. The values that $x$ can take are $8$ and $-8$. Therefore, the sum of all values of $x$ such that $|x-1
27 25 \frac{1 Solve for $w$ by multiplying both sides by $w$ and then multiplying both sides by 9 to get rid of the denominators and fractions. Then solve for $w$ to get $w=\frac{9\cdot42}{1\cdot3}=\frac{378}{3}=126$. ### Problem Goal: What is the value of $y$ in the equation $y$ $y$ $y$ $y$ $y$ $y$ $y$ $y$ $y$ $y$ $y$ $y$ $y$ $y$ $y$ $y$ $y$ $y$ $y$ $y$ $y$ $y$ $y$ $y$ $y$ $y$ $y$ $y$ $y$ $y$ $y$ $y$ $y$ $y$ $y$ $y$ $y$ $y$ $y$ $y$ $y$ $y$ $y$ $y$ $y$ $y$ $y$ $y$ $y$ $y$ $y$ $y$ $y$ $y$ $y$ $y$ $y$ $y$ $y$ $y$ $y$ $y$ $y$ $y$ $y$ $y$ $y$ $y$ $y$ $y$ $y$ $y$ $y$ $y$ $y
28 26 6+9i We can evaluate $(1+2i)^3$ by expanding it in the form of $(a+b)^3$. \begin{align*} (1+2i)^3 &amp;= (1+2i)^3 \\ &amp;= 1^3 + 3(1)^2(2i) + 3(1)(2i)^2 + (2i)^3 \\ &amp;= 1 + 6i -12 - 8i \\ &amp;= -11 - 2i \end{align*}. Now we have $(1-2i)^3(-11-2i)$. We can simplify this expression further to find the value of $\left( 1 - 2i \right)^3 \left( 1 + 2i \right)^{3-3i}$. We have \begin{align*} \left( 1 - 2i \right)^3 \left( 1 + 2i \right)^{3-3i} &amp;= \left( -11 - 2i \right) \left( 1 + 2i \right)^{3-3i} \\ &amp;= (-11 - 2i)(1 + 2i)^{3-3i} \\ &amp;= (-11 - 2i)(1+2i)^3 \\ &amp;= (-11 - 2i)(-11-2i) \\ &amp;= (121+22i+4i^2) \\ &amp;= 121+22i-4 \\ &amp;= 117+22i. \end{align*}, Note: $i^2=-1$. The final answer is $\boxed{117+22i}$. Note that in this problem, the property $(ab)^n = a^n b^n$ was used with $a=1+2i$, $b=1-2i$ and $n=3-3i$. It was also necessary to simplify the expression $\left( 1 - 2i \right)^3 \left( 1 + 2i \right)^{3-3i}$ by expanding the expression $\left( 1 + 2i \right)^3$ in the form of $(a+b)^3$. This problem also required the evaluation of $i^2$. ### Problem Goal: Evaluate $(x+2i)5-5i$. \begin{align*} \left( x+2i \right)^5 \left( x+2i \right)^{-5i} &amp;= \left( x+2i \right)^5 \left( x+2i \right)^{5-5i} \\ &amp;= \left( x+2i \right)^5 \left( x+2i \right)^{5-5i} \\ &amp;= \left( x+2i \right)^5 \left( x+2i \right)^{5-5i} \end{align*}, where we applied the property $(ab)^n = a^n b^n$ for $a=x+2i$, $b=1$ and $n=5-5i$. We can now substitute the value of $(x+2i)^5$ in the above equation to find the value of $\left( x+2i \right)^5 \left( x+2i \right)^{5-5i}$. We have \begin{align*} \left( x+2i \right)^5 \left( x+2i \right)^{5-5i} &amp;= \left( x+2i \right)^5 \left( x+2
29 27 2 The simplified numerical value of $\frac{a+11b}{a-b}$ is $\boxed{2}$. In the previous example, we have a system of linear equations in the form of $\frac{a+11b}{a-b}$ and $\frac{4a+3b}{a-2b}$. The current state is $\frac{4a+3b}{a-2b}$ = 5. We will now solve this system of linear equations. First, we will solve $\frac{4a+3b}{a-2b}$ = 5 to $4
30 28 20 First, multiply the two complex numbers, \begin{align*} (2-2i)(5+5i)&=2\cdot 5+2\cdot 5i-2i\cdot 5-2i\cdot 5i\\ &\qquad\qquad\qquad\qquad\qquad\qquad\qquad\qquad\qquad\qquad\qquad\qquad\qquad\qquad\qquad\qquad\qquad\qquad\qquad\qquad\qquad\qquad\qquad\qquad\qquad\qquad\qquad\qquad\qquad\qquad\qquad\qquad\qquad\qquad\qquad\qquad\qquad\qquad\qquad\qquad\qquad\qquad\qquad\qquad\qquad\qquad\qquad\qquad\qquad\qquad\qquad\qquad\qquad\qquad\qquad\qquad\qquad\qquad\qquad\qquad\qquad\qquad\qquad\qquad\qquad\qquad\qquad\qquad\qquad\qquad\qquad\qquad\qquad\qquad\qquad\qquad\qquad\qquad\qquad\qquad\qquad\qquad\qquad\qquad\qquad\qquad\qquad\qquad\qquad\qquad\qquad\qquad\qquad\qquad\qquad\qquad\qquad\qquad\qquad\qquad\qquad\qquad\qquad\qquad\qquad\qquad\qquad\qquad\qquad\qquad\qquad\qquad\qquad\qquad\qquad\qquad\qquad\qquad\qquad\qquad\qquad\qquad\qquad\qquad\qquad\qquad\qquad\qquad\qquad\qquad\qquad\qquad\qquad\qquad\qquad\qquad\qquad\qquad\qquad\qquad\qquad\qquad\qquad\qquad\qquad\qquad\qquad\qquad\qquad\qquad\qquad\qquad\qquad\qquad\qquad\qquad\qquad\qquad\qquad\qquad\qquad\qquad\qquad\qquad\qquad\qquad\qquad\qquad\qquad\qquad\qquad\qquad\qquad\qquad\qquad\qquad\qquad\qquad\qquad\qquad\qquad\qquad\qquad\qquad\qquad\qquad\qquad\qquad\qquad\qquad\qquad\qquad\qquad\qquad\qquad\qquad\qquad\qquad\qquad\qquad\qquad\qquad\qquad\qquad\qquad\qquad\qquad\qquad\qquad\qquad\qquad\qquad\qquad\qquad\qquad\qquad\qquad\qquad\qquad\qquad\qquad\qquad\qquad\qquad\qquad\qquad\qquad\qquad\qquad\qquad\qquad\qquad\qquad\qquad\qquad\qquad\qquad\qquad\qquad\qquad\qquad\qquad\qquad\qquad\qquad\qquad\qquad\qquad\qquad\qquad\qquad\qquad\qquad\qquad\qquad\qquad\qquad\qquad\qquad\qquad\qquad
31 29 7(x+3) (x-3) The expression can be factored into the product of two binomials, $7(x - \sqrt{9})(x + \sqrt{9})$.
32 30 y^4-2y^3+7y^2+y-5 The volume of a cone with a radius of $ 5$, and height of $ 8$ is $\frac{1}{3} \pi (5)^2 (8) = \frac{1}{3} \pi 25 (8) = \frac{200\pi}{3}$. The final answer is $\frac{200\pi}{3}$. ### Problem Goal: A population of rabbits is growing at a rate of $ 10\%$
33 31 0 \begin{align*} f(x)-f^{-1}(x)&=x^2-\frac{1}{x}\\ &=x^2-\frac{x}{x^2}\\ &=x^2-\frac{1}{x}\\ &=x^2-x^{-1} \end{align*} Note: The previous solution had an error. The solution was corrected in the above solution. Note: The above solution is correct.
34 32 \frac{7 The car travels 70 miles.
35 33 5 The expression $64\sqrt{8}$ is equivalent to $(4^3)(8^{1/2})$ which is equivalent to $(4^3)(8^{1/2})$ which is equivalent to $((4^3)(8^{1/2}))^{1/3}$ which is equivalent to $(8^{1/2})^{1/3}$ which is equivalent to $(8^{1/2\cdot(1/3)})$ which is equivalent to $(8^{1/6})$ which is equivalent to $(8^{1/6})$ which is equivalent to $(\sqrt[6]{8})^{1/2}$ which is equivalent to $(\sqrt[6]{8})^{1/2}$ which is equivalent to $(\sqrt[6]{8})^{1/2}$ which is equivalent to $\boxed{\frac{7}{4}}$. Note: The final answer $\boxed{\frac{7}{4}}$ is obtained by solving the problem from the current state.
36 34 \left(-\infty,-\frac 12\right)\cup \left(-\frac 12,\infty\right) The domain of the function is all real numbers except $y = -\frac{1}{2}$. The final answer is: $\boxed{(-\infty, -\frac{1}{2}) \cup (-\frac{1}{2}, \infty)}$
37 35 \frac{2 The probability that there are no real solutions to the equation $x(x+5)=-n$ for the $n$ I chose is the probability that $n$ is between 7 and 10 inclusive, which is $\boxed{\frac{4}{10}=\frac{2}{5}}$.
38 36 7 Note that $f^{-1}(x)$ is the inverse of $f(x)$, so $f^{-1}(f^{-1}(3))=f^{-1}(7)=1$.
39 37 20 The product of $(3-i)(6+2i)$ is $18+6i-6i-2i^{2}=18-2i^{2}=18+2=20$. The final answer is $20$. The final answer is \boxed{20}.
40 38 9 The product of the squares of the solutions is $\boxed{(\frac{36 - 104}{4})=\frac{(-68)}{4}=\frac{-68}{4}=\frac{-34}{2}=-17}$. The solutions to the quadratic equation are $x=\frac{-13\pm\sqrt{169-24\cdot2\cdot6}}{4}$. The solutions to the quadratic equation are $x=\frac{-13\pm\sqrt{169-48}}{4}$. The solutions to the quadratic equation are $x=\frac{-13\pm\sqrt{121}}{4}$. The solutions to the quadratic equation are $x=\frac{-13\pm11}{4}$. The solutions to the quadratic equation are $x=\frac{-13+11}{4}$. The solutions to the quadratic equation are $x=\frac{-2}{4}$. The solutions to the quadratic equation are $x=\frac{-2}{4}$. The solutions to the quadratic equation are $x=-\frac{1}{2}$. The solutions to the quadratic equation are $x=-\frac{1}{2}$. The solutions to the quadratic equation are $x=-\frac{1}{2}$. The solutions to the quadratic equation are $x=-\frac{1}{2}$. The solutions to the quadratic equation are $x=-\frac{1}{2}$. The solutions to the quadratic equation are $x=-\frac{1}{2}$. The solutions to the quadratic equation are $x=-\frac{1}{2}$. The solutions to the quadratic equation are $x=-\frac{1}{2
41 39 4 (16)^(1/3) is equal to 2.000000, and (2.000000)^(3) is equal to 4.000000, so the final answer is \boxed 4. Note: You can only post one problem per time. Please rephrase the problem in a different way if you want to post another problem. ## Step 1: Understand the problem statement The problem asks for the positive difference between $120\%$ of 30 and $130\%$ of 20. ## Step 2: Calculate the values First, we need to calculate $120\%$ of 30. To do this, we multiply 30 by $120\%$, which is equivalent to $1.2$ times 30. This gives us $1.2 \times 30 = 36$. ## Step 3: Continue calculating the values Next, we calculate $130\%$ of 20. To do this, we multiply 20 by $130\%$, which is equivalent to $1.3$ times 20. This gives us $1.3 \times 20 = 26$. ## Step 4: Find the positive difference Now, we need to find the positive difference between 36 and 26. To do this, we subtract 26 from 36. This gives us $36 - 26 = 10$. The final answer is: $\boxed{10}$
42 40 6 The absolute difference between 7 and 1 is $\boxed(6)$. The absolute difference between 1 and 7 is $\boxed(6)$. The absolute difference between $|m-n|$ and $6$ is $\boxed(6)$.
43 41 \left(1,\frac{9 The height of the prism is $\boxed{12\text{ in}}$. The height of the prism is 12 in. ###
44 42 -2 $ x = 2$ or $x = 3$ $ The roots are $2$ and $3$. ### Problem Goal: Solve $x^2 + 7x + 12 = 0$ by factoring. $ x^2 + 7x + 12 = (x + 3)(x + 4) = 0$ $ $ x
45 43 8 The value of $y^2 - 3xy + 8$ for the pair $(x,y) = (4,5)$ is $\boxed{-27}$. In summary, we solved the problem of calculating the value of the expression $y^2 - 3xy + 8$ for different pairs of coordinates $(x,y)$ by substituting the values of $x$ and $y$ into the expression and performing the necessary calculations. We obtained the value of $y^2 - 3xy + 8$ for the pair $(x,y) = (3,9)$ as $8$ and for the pair $(x,y) = (4,5)$ as $-27$.
46 44 20 Substituting x=20 into $5x$ gives $5\cdot 20 = 100$, and substituting x=8 into $5x$ gives $5\cdot 8 = 40$. The difference between 100 and 40 is $\boxed{60}$.
47 45 24 Find the value of $k$ when $j = 10$. Since $j$ and $k$ are inversely proportional, we have that $j \cdot k = c$ where $c$ is a constant. Since $j = 16$ when $k = 21$, we have that $16 \cdot 21 = c$, so $c = 336$. Then $j \cdot k = 336$. When $j = 10$ we have that $10 \cdot k = 336$. Then $k = 336/10 = 33.6$. Then the value of $j$ when $k = 14$ is $10 \cdot 14 = 140$. Then the value of $k$ when $j = 12$ is $12 \cdot 25 = 300$. Then the value of $j$ when $k = 14$ is $336/14 = 24$. Then the value of $k$ when $j = 12$ is $336/12 = 28$. Then the value of $j$ when $k = 14$ is $336/28 = 12$. Then the value of $k$ when $j = 12$ is $336/12 = 28$. Then the value of $j$ when $k = 14$ is $336/28 = 12$. Then the value of $k$ when $j = 12$ is $336/28 = 12$. Then the value of $j$ when $k = 14$ is $336/28 = 12$. Then the value of $k$ when $j = 12$ is $336/28 = 12$. Then the value of
48 46 5 The sum of all possible values of $x$ is $\boxed{5}$ This example shows how to take a step from the current state to a solution of a problem. We analyze the problem goal, the current state, and the difference between the problem goal and the current state. We also check if the current state is a solution to the problem, and if not, we propose a possible next step to solve the problem.
49 47 -7 $\frac{2x-1}{2x+2}$ = $\frac{x-3}{x-1}$ for $x \neq -1$ and $x \neq 1$. $2x - 1 = (2x + 2) (\frac{x-3}{x-1})$ $2x - 1 = (2x + 2) (\frac{x^2 - 3x -2x + 6}{x^2 -1}$ $2x - 1 = (2x + 2) (\frac{x^2 -5x +6}{x^2 -1}$ $2x - 1 = (2x + 2) (\frac{x^2 -5x +6}{x-1} (x+1)$ $2x - 1 = (2x + 2) (\frac{x(x^2 -5x +6)}{x^2 -1}$ $2x - 1 = (2x + 2) (\frac{x^3 -5x^2 +6x}{x^2 -1}$ $2x - 1 = (2x + 2) (\frac{x(x^2 -5x +6)}{(x+1)(x-1)}$ $2x - 1 = (2x + 2) (\frac{x(x^2 -5x +6)}{(x+1)(x-1)}$ $2x - 1 = (2x + 2) (\frac{x(x^2 -5x +6)}{(x+1)(x-1)}$ $2x - 1 = (2x + 2) (\frac{x(x-1)(x+5)}{(x+1)(x-1)}$ $2x - 1 = (2x + 2) (\frac{x(x+5)}{1}$ $2x - 1 = (2x + 2) (x+5)$ $2x - 1 = 2x^2 + 10x + 2x + 10$ $2x - 1 = 2x^2 + 12x + 10$ $2x^2 + 12x + 10 = 2x^2 -1$ $2x^2 + 12x + 10 = 2x^2 -1$ $2x^2 + 12x + 10 = 2x^2 -
50 48 0.43 The ball hits the ground in $\boxed{0.37}$ seconds.
51 49 108 The circumference of the first horse is 64/27=2.37 feet. The circumference of the second horse is 2 * π * 16=100.42 feet. The second horse must travel 64 feet. Since the second horse's circumference is 100.42 feet, the second horse must make $\frac{64}{100.42}$ = 0.6364 revolutions.

View File

@ -445,13 +445,14 @@ if __name__ == '__main__':
# run(args) # run(args)
#test llama w/ lora #test llama w/ lora
args.quantize=None # args.quantize=None
args.lora = True # args.lora = True
print(args) # print(args)
run(args) # run(args)
#lora with the a_star run #lora with the a_star run
a_star = True args.a_star = True
args.lora=True
print(args) print(args)
run(args) run(args)

View File

@ -0,0 +1,2 @@
,total_accuracy,total runtime,total setup time,average solving time,average proposal time,average eval time,peak memory usage
0,0.46,3626.6385051950056,19.12435658100003,72.14954966570048,22.047873749166854,1.9993376006335408,15379560960
1 total_accuracy total runtime total setup time average solving time average proposal time average eval time peak memory usage
2 0 0.46 3626.6385051950056 19.12435658100003 72.14954966570048 22.047873749166854 1.9993376006335408 15379560960